Kelly has the following income and expenses:
$200 per month paychecks
$50 cell phone
$40 gasoline
$40 Miscellaneous expenses
How much money does Kelly have remaining after paying her monthly expenses?

Answers

Answer 1
Just add the amount of her expenses
50+40+40=130 and then subtract that from her pay 200-130= $70
So she has $70
Hope this helps :)

Related Questions

PLEASE HELP
Simplify the expression 4x³ × 2x³
3
A) 8x^6
B) 8x⁹
C)2x^6
D) 2x⁹

Answers

Answer:it isA

Step-by-step explanation:

Answer:

8x^6

Step-by-step explanation:

Add the exponents and multiply the numbers

Show that the set of numbers in the interval [0,1] having decimal expansions using only odd digits is closed. Describe this set by a Cantor set type construction.

Answers

T be the set of all points x in [0,1] by a Cantor set type construction is  x=∑∞n=1 qn/4n

for some sequence of qns with each qn∈{1,3}.

What is Cantor set type construction?

The Cantor set is a collection of points situated on a single line segment that possesses a variety of odd characteristics.

By deleting the center third of a line segment and then repeating the process with the remaining shorter segments, the Cantor ternary set, the most popular construction, is created. The ternary construction was only briefly described by Cantor as an illustration of a more basic concept—that of a perfect set that isn't dense anywhere.

According to the given question:

The proportion (i.e., measure) of the unit interval that is left can be determined by subtracting the entire length as the Cantor set is defined as the set of points that are not excluded.

T be the set of all points x in [0,1] by a Cantor set type construction is  x=∑∞n=1 qn/4n

for some sequence of qns with each qn∈{1,3}.

TO learn more about cantor set visit:

brainly.com/question/17094634

#SPJ4

3) Lourdes got a plant as a birthday gift. Her plant grows an average of 1 2/5 inches every month. How long will it take for the plant to grow a full 10 2/3

Answers

Answer:

7 13/21 months

Step-by-step explanation:

10 2/3 ÷ 1 2/5 =

= 32/3 ÷ 7/5

= 32/3 × 5/7

= 160/21

= 7 13/21

Answer: 7 13/21 months

We can treat this like an equation.
We know the rate per month (1 2/5) and the end result (10 2/3).
X is the number of months, bc we don’t know how LONG it’s gonna take.

So 1 2/5 * x = 10 2/3

Now to solve this we have to first change the numbers to improper fractions. 1 2/5 becomes 7/5 and 10 2/3 becomes 32/3.

So now, 7/5 x = 32/3

To figure out what x is we have to isolate it. To do that when it’s connected to a fraction, we have to multiply both sides by the reciprocal of that fraction, which is 5/7. That cancels out the fraction.

So now, x = 160/21
Simplified that means x = 7 13/21
So it’s going to take the plant 7 13/21 months to grow a full 10 2/3.
This looks like a weird number, but my math looks right, so if I’m wrong correct me but hope this helps.

start by thinking of the number 22 as divided into 22 individual items and the variables x1, x2, and x3 as three categories into which these items are placed. since each xi is a positive correct: your answer is correct. integer, start by placing one item in each category and distribute the remaining items among the categories. the number of solutions to the equation that satisfy the given condition is the number of ways to place all the items into the categories. thus, the answer is 20 incorrect: your answer is incorrect. .

Answers

The number of solution to the equation is 210 when consider the number 22 as being composed of 22 distinct items, with the variables x₁, x₂ and x₃.

Given that,

Consider the number 22 as being composed of 22 distinct items, with the variables x₁, x₂ and x₃ . acting as the three categories into which these items are sorted. Since each x is a true affirmative, your response is accurate. Put one item in each category to begin with, then divide the remaining items across the categories. The number of classification schemes for all the objects is the number of solutions to the equation satisfying the given criteria. In light of this, your response of 20 is erroneous.

We know that,

Number 22 is divided into 22 individual items ,

The three categories in which these items are placed are x₁, x₂ and x₃ .

x₁+x₂+x₃=22----->equation(1)

(x₁-1)+(x₂-1)+(x₃-1)=22-3=19

Let

a=x₁-1,

b=x₂-1 and

c=x₃-1.

So,

a+b+c= 19 ----->equation(2)

So , the number of ways of arranging 21 characters in which 19 are same(A) and 2 are same (J)

= 21!/(2! × 19!)

= 21×10 = 210

Therefore, the number of solution to the equation is 210 when consider the number 22 as being composed of 22 distinct items, with the variables x₁, x₂ and x₃.

To learn more about solutions visit: https://brainly.com/question/28705452

#SPJ4

a nail is measured with a ruler and its length is reported as 5.46 cm. select all the statements that are correct about the certainty and uncertainty represented in this recorded measurement.

Answers

If the nail is measured with a ruler and its length is 5.46 cm , then the correct statements are :

(a) The uncertainty can be expressed by reporting the measurement as 5.46 ± 0.1 cm ;

(b) The uncertainty is in the final digit of the measurement .

The Uncertainty of a number helps to estimates in comparison of experimental numbers .

it is given that , the reported length of the ruler is = 5.46 cm ;

we know that ,

the uncertainty In any number , is present in last digit only ,

the place value of the last digit in 5.46 is 0.01 .

So , uncertainty can be expressed by reporting measurement as 5.46 ± 0.1 cm

Therefore , the correct statements about the certainty and uncertainty are (a) and (b) .

The given question is incomplete , the complete question is

A nail is measured with a ruler and its length is reports as 5.46 cm.

Select ALL the statements that are correct about the certainty and uncertainty represented in this recorded measurement.

(a) The uncertainty can be expressed by reporting the measurement as 5.46 ± 0.1 cm ,

(b) The uncertainty is in the final digit of the measurement ,

(c) All three digits shown in the measurement are known with certainty ,

(d) The uncertainty can be expressed by reporting the measurement as 5.46 ± 0.01 cm  .

Learn more about Uncertainty here

https://brainly.com/question/28351631

#SPJ4

Calculate the area, in square units, bounded by f(x) = 3x^3-3x^2-x+8 and g(x) = 2x^3 - 10x^2 + 29x + 8 over the interval [1, 5].

Answers

The area bounded by f(x)=3x³-3x²-x+8, g(x) = 2x³- 10x² + 29x+8

over the interval [1, 5] is 164 square units.

What is meant by a function?

Exactly one element of Y is assigned to each element of X via a mathematical function from a set X to a set Y. The domain and codomain of the function are, respectively, the sets X and Y as a whole.

The first presentation of the idea of function that is known about was developed by two Persian mathematicians, Al-Biruni and Sharaf al-Din al-Tusi. Originally, functions were used to depict the desired relationship between two varying values. For example, time affects the position of a planet. Up to the 19th century, the functions that were taken into consideration were differentiable.

Given,

f(x)=3x³-3x²-x+8

g(x) = 2x³- 10x² + 29x + 8

And the interval is  [1, 5].

By using area formula,

A=[tex]\int\limits^a_b {f(x)-g(x)} \, dx[/tex]

A=[tex]\int\limits^a_b[/tex](Upper function-Lower function)dx

A=[tex]\int\limits^3_1[/tex][g(x)-f(x)]dx+[tex]\int\limits^5_3[/tex][f(x)-g(x)]dx

A=[tex]\int\limits^3_1[/tex](-x³-7x²+30x)dx+[tex]\int\limits^5_3[/tex](x³+7x²-30x)dx

A= (118/3)+(374/3)

A=164 square units.

Therefore, the area bounded by f(x)=3x³-3x²-x+8, g(x) = 2x³- 10x² + 29x+8

over the interval [1, 5] is 164 square units.

To know more about function, visit:

https://brainly.com/question/28278699

#SPJ1

1. Assume there are 5 red balls, 6 blue balls, and 4 green balls. If the balls are removed from the box one at a time, in how many different orders can the balls be removed assuming two balls of the same type are indistinguishable.
2. Give a recursive definition of the set of all even positive integers not divisible by 3.
Please write clearly so I can study from it! Try not to skip steps as much as you can.
Thank you!

Answers

There are 1307674368000 possible methods to remove each ball from the box one at a time.

what is probability?

How likely something is to occur is known as its probability. We can discuss the probabilities of various outcomes—how likely they are—when we aren't sure how a particular event will turn out.  examination of events subject to probability.

given,

the quantity of balls inside a box ,

four red balls

6 balls in blue

four green balls

The variety of ways the balls can be taken out of the box one at a time must be determined.

Here,

Combination is possible because order is imperceptible;

mixture formula; nCr is equal to n!/r1(n-r)!

Here, There are 15 balls in the box overall (5 + 4 + 6 balls).

Then.

¹⁵C₁₅ = 15! / 15! (15 - 15)! = 15! = 1307674368000

that is

There are 1307674368000 ways in which we can take the balls out of the box one at a time.

To know more about probability visit:-

https://brainly.com/question/11234923

#SPJ4

Find the area of the shape.
Either enter an exact answer in terms of \piπpi or use 3.143.143, point, 14 for \piπpi and enter your answer as a decimal.

Answers

Answer:

[tex]\pi[/tex]

Step-by-step explanation:

Area = [tex]\pi x^{2}[/tex]  For a full circle.  This is 1/4 of a circle

A= [tex]\frac{\pi 2^{2} }{4}[/tex] = [tex]\frac{\pi 4}{4}[/tex] = [tex]\pi[/tex]

Answer : The area of the shape is, 111.02 units 2

For each of the values of GPD, use the regression equation to predict the carbon dioxide emission amount. Type either a numerical value or the word no. Do not round the answer. Units are in parentheses, so do not convert your answer 2.6 (trillions of dollars) (millions of metric tons) 4.0 (trillions of dollars) (millions of metric tons) 7.1 (trillions of dollars) (millions of metric tons

Answers

For each of the values of GPD, use the regression equation to predict the carbon dioxide emission amount is 1300.715.

What is carbon dioxide ?

Our atmosphere contains carbon dioxide, an odorless and colorless gas. According to its chemical composition, CO2, it is made up of one carbon atom joined to two oxygen atoms. It is a waste product created both by burning fossil fuels and by human bodies.

Given regression eq.

[tex]$$\hat{y}=166.92+115.725$$[/tex]

* Given [tex]$x=2.6$[/tex]

[tex]$$\begin{aligned}& \hat{y}=166.9(2.6)+115.725 \\& \hat{y}=549.665\end{aligned}$$[/tex]

* Given [tex]$x=4.0$[/tex]

[tex]$$\begin{aligned}& \hat{y}=166.9(4.0)+115.725 \\& \hat{y}=783.325\end{aligned}$$[/tex]

* Given [tex]$x=7.1$[/tex]

[tex]$$\hat{y}=166.9(7.1)+115.725$$[/tex]

[tex]\hat{y}=1300.715[/tex]

To learn more about carbon dioxide visit:https://brainly.com/question/3049557

#SPJ4

Prove that a polynomial function f of odd degree has at least one real root. Hint: it may help to consider first the case of a cubic f(x)=a0+a1x+a2x^2+a3x^3.

Answers

the Discriminant will always be greater than or equal to zero. This means that the polynomial has at least one real root.

Let f(x) = a0 + a1x + a2x2 + a3x3 be a polynomial function of odd degree 3.

We know that the degree of a polynomial is the highest power of the variable in the equation. Since the degree of f(x) is 3, then a3 ≠ 0.

Now, consider the Discriminant of the polynomial, which is defined as the expression below:

Discriminant = b2 - 4ac

In this case, b2 = a2^2, c = a0 and a = a3, thus the Discriminant is:

Discriminant = a2^2 - 4a0a3

Since a3 ≠ 0 (as mentioned before), the Discriminant will always be greater than or equal to zero. This means that the polynomial has at least one real root.

Learn more about polynomial here

https://brainly.com/question/11536910

#SPJ4

Find the value of x in the figure
Please show all your work.

Answers

Answer:

x = 7/3.

Step-by-step explanation:

DE is parallel to BC and AD= DB and AD = EC.

So, AD/DB = (x+1)/(4x-6)

(x + 1)/(4x - 6) = 1

x + 1 = 4x - 6

1 + 6 = 4x - x

3x = 7

x = 7/3.

Pls help its due today!!! 50 points!!!

Answers

Answer:

Step-by-step explanation:

Part 1: true

(-4, -1) and (0, 2)

m = 2+1/0+4

m = 3/4

(0, -3) and (4, 0)

m = 0+3/4+0

m = 3/4

Part 2: true

It intercepts at (0, -3)

Part 3:

I can't really see what slope it says but if it says -3/5, then its true, otherwise it's false.

(5, -1) and (0, 2)

m = 2+1/0-5

m = 3/-5

m = -3/5

Part 4: false

Graph A would have 2 and Graph C would have different intersecting points.

Which expression is equivalent to 3√x^5y^?

Answers

The equivalent expression for the given expression is [tex]x^{\frac{5}{3} }y^{\frac{1}{3} }[/tex]. Therefore, option B is the correct answer.

What is an equivalent expression?

Equivalent expressions are expressions that work the same even though they look different. If two algebraic expressions are equivalent, then the two expressions have the same value when we plug in the same value for the variable.

The given expression ∛x⁵y.

expression is equivalent to ∛x⁵y is

[tex](x^5y)^{\frac{1}{3} }[/tex]

= [tex]x^{\frac{5}{3} }y^{\frac{1}{3} }[/tex]

The equivalent expression is [tex]x^{\frac{5}{3} }y^{\frac{1}{3} }[/tex]. Therefore, option B is the correct answer.

To learn more about an equivalent expression visit:

https://brainly.com/question/28170201.

#SPJ1

Let X1,X2,...,Xn be a random sample of size n from a geometric distribution for which p is the probability of success.
(a) Use the method of moments to find a point estimate for p.
(b) Explain intuitively why your estimate makes good sense.

Answers

a) X1+X2 have distribution Bi(n1+n2, p)

b) P(X1 + X2 = 1) = (1-p)ⁿ¹ * np(1-p)ⁿ²⁻¹+ (1-p)ⁿ²*np(1-p)ⁿ¹-¹

Since both variables are independent but they have the same probability parameter, you can interpret that like if the experiment that models each try in both variables is the same. When you sum both random variables toguether, what you obtain as a result is the total amount of success in n1+n2 tries of the same experiment, thus X1+X2 have distribution Bi(n1+n2, p).

b)

Note that, if X2 = k, then X1+X2 = 1 is equivalent to X1 = 1-k. Since X1 and X2 are independent, then P(X1+X2 = 1| X2 = K) = P(X1=1-k|X2=k) = P(X1 = 1-k).

If k = 0, then this probability is equal to P(X1 = 1) = np(1-p)ⁿ¹⁻¹

If k = 1, then it is equal to P(X1 = 0) = (1-p)ⁿ¹

Thus,

P(X1+X2 = 1) = P(X1+X2 = 1| X2 = 1) * P(X2=1) + P(X1+X2 = 1| X2 = 0) * P(X2 = 0) = (1-p)ⁿ¹ * np(1-p)ⁿ²⁻¹+ (1-p)ⁿ²*np(1-p)ⁿ¹-¹

learn more about of distribution here

https://brainly.com/question/14690794

#SPJ4

What is 7/2 with a denominator of 10

Answers

Answer: 35/10

Step-by-step explanation:

if you have 7/2 and need a denominator of 10 all you need to do is multiply 7/2 by 5/5.

7  by  5     35

2  by  5     10

The value of 7/2 with a denominator of 10 is 35/10

How to rationalize a fraction?

Suppose the given fraction is [tex]\dfrac{a}{b+c}[/tex]

Then the conjugate of the denominator is given by b - c

Thus, rationalizing the fraction will give us

[tex]\dfrac{a}{b+c} \times \dfrac{b-c}{b-c} = \dfrac{a(b-c)}{b^2 - c^2}[/tex]

(we used the identity [tex](b-c)(b+c) = b^2 - c^2 )[/tex]

We actually rationalize just for the use of that later denominator or numerator

We have 7/2 and need a denominator of 10 all you need to do is multiply 7/2 by 5/5.

7 × 5 = 35

2 × 5 = 10

Learn more about rationalization here;

https://brainly.com/question/14261303

#SPJ2

what is the width of the frame?

Answers

the horizontal with of the frame and is measured from the outmost side of the frame.

592 x 43 in standard algorithm

Answers

Answer:

25456

Step-by-step explanation:

592 over 43

592

x 43

multiply 3 by 2, 9 and 5. 3 by 2 is 6. 3 by 9 is 27. 3 by 5 is 15.

add a zero at the second row then,

multiply 4 by 2, 9, 5. 4 by 2 is 8. 4 by 9 is 36. 4 by 5 is 20.

if you aligned them properly, when you add them you should get 25456

I'll try to include a picture of my work, hopefully you can see it**

Find the standard deviation for each data set. Use the standard deviations to compare the pair of data sets. fastest recorded speeds of various large wild cats (miles per hour): 70 45 25 45 35 35 40 40 25 fasted recorded speeds of various birds in flight (miles per hour): 216 103 96 56 63 35 50 32 55 25 25 20

The standard deviation of the large wild cats is ō=
The standard deviation of the birds in flight is ō=
The standard deviations of the two data sets show that the fastest recorded speeds of large wild cats deviate _____ from the mean than the fastest recorded speeds of birds in flight.

A. Less
b. more

Answers

(a) The standard deviation of the large wild cats is 25.5 (b)  The standard deviation of the birds in flight is 33 (c) The standard deviations of the two data sets show that the fastest recorded speeds of large wild cats deviate  more from the mean than the fastest recorded speeds of birds in flight . Option A

How to determine the standard deviation?

We should understand that standard deviation looks at the amount of deviation from the means of the set of data.

(a) (70+45+25+45+35+35+40+40+25)÷9 = 40

mean is = 40

(x-m) =30+5+-15+5+-5+-5+0+0+-15)

(x-m)²=(900+25+225+25+25+25+0+0+225)=1450/9

Standard deviation = √650

Standard deviation = 25.5

(b) x= (216+103+96+56+63+35+50+32+55+25+25+20)=776

Mean(m)=776/12=65

(x-m)=151+38+31+-9+-2+-30+-15+-33+-10+-40+-40+-45

Now the square of the deviations from the mean is

(x-m)²=(7396+1444+961+81+4+900+225+1089+100+1600+1600+2025)÷12

=1452

Standard deviation = √1452=33

(c) Therefore, from the calculations, the standard deviations of the two data sets show that the fastest recorded speeds of large wild cats deviate more from the mean than the fastest recorded speeds of birds in flight

Learn more about standard deviation on https://brainly.com/question/23907081

#SPJ1

6x+6y=90 9x+2y=79 What pair of numbers solves the system?​

Answers

Answer:

(7, 8)

Step-by-step explanation:

equation 1:  [tex]6x+6y=90[/tex]

equation 2: [tex]9x+2y=79[/tex]

This system of equations can be solved for using the elimination method.

First, multiply both sides of the first equation by 3/2 to match the number of x's in both equations so that when we subtract the first equation from the second, the x's cancel and we can solve for y.

[tex]\left(\dfrac{3}{2}\right)\left(6x+6y\right)=(90)\left(\dfrac{3}{2}\right)[/tex]

[tex]9x + 9y = 135[/tex]

Next, subtract this from the second equation.

    [tex]9x+2y=79[/tex]       ← eq. 2

[tex]\underline{- (9x+9y=135)}[/tex]    ← modified eq. 1

     [tex]0 - \, 7y \, = -56[/tex]

Then, solve for y by dividing by 7.

[tex]7y = 56\\\overline{\, 7 \ } \ \ \ \ \overline{\ 7 \: }[/tex]

[tex]y=8[/tex]

So, the y-coordinate of the solution pair is 8.

Now that we've solved for the y, all we have to do to get x is plug the solved y-value back into one of the equations. I'll use the first equation.

[tex]6x+6y=90[/tex]

[tex]6x + 6(8) = 90[/tex]

↓ multiplying out 6 and 8

[tex]6x + 48 = 90[/tex]

↓ subtracting 48 from both sides

[tex]6x = 90 - 48[/tex]

[tex]6x = 42\\\overline{\: 6 \: } \ \ \ \ \, \overline{\: 6 \: }[/tex]

[tex]x = 7[/tex]

So, the x-coordinate of the solution pair is 7.

Finally, put the x- and y-coordinates together into an ordered pair in the form [tex](x, y)[/tex].

(7, 8)

Answer:

(7, 8) is the required pair of numbers

---------------------------

Given system

6x + 6y = 90, 9x + 2y = 79.

Solve it by substitution

First, simplify the first equation by dividing all terms by 6:

x + y = 15

Then solve it for y:

y = 15 - x

Now, substitute this into the second equation:

9x + 2(15 - x) = 799x + 30 - 2x = 797x = 79 - 307x = 49x = 7

Finally, find the value of y:

y = 15 - 7y = 8

The pair of numbers is (7, 8).

Suppose you observed the following sample:
6.2, 7.4, 9.6, 11.5, 13.5
Compute the 2nd sample moment. (please type in the exact number, do not round your answer)

Answers

449.86

To compute the moment of the second sample, we need to compute the sum of the squares of the sample values.

This formula is:

Sum = ∑(x_i^2)

After inserting the example values:

Sum = 6.2^2 + 7.4^2 + 9.6^2 + 11.5^2 + 13.5^2

Simplifying this formula to:

Total = 38.44 + 54.76 + 92.16 + 132.25 + 181.25

Adding these terms gives us the following final result:

Total = 449.86

Learn more about sample moment on https://brainly.com/question/14987800

#SPJ4

If i have 2 apples and i get 69 times that, than i give three-ninths of it to a friend, how much would i have left?

Answers

Answer:

92 apples remaining

Step-by-step explanation:

2 * 69 = 138 apples

138 * (3/9) =

138 * (1/3) =

138 * 1/3 =

138/3 = 46 apples

138 - 46 = 92 apples remaining

Answer:

you have 2 apples

then you got it 69 times= 2×69 = 138

then you give 3 ninths to a friend

138/9 = 15.33

you gave it 3 one-ninths = 3× 15.33 = 46

so must be left with 138-46 = 92

(c) Solve by Completing the Square (Round answers to the nearest tenth)
x²+10 x+2=2 x+5

Answers

After solving the equation by Completing the square , the answer is x = 0.4 , x = -8.4  .

In the question ,

it is given that ,

the equation is x² + 10x + 2 = 2x + 5,

rewriting the equation ,

we get ,

x² + 8x - 3 = 0

adding 3 to both sides ,

we get ,

x² + 8x = 3 ,

For the method of completing the square ,

we take half of the second term's coefficient, and square it, then add it to both sides.

that is 8/2 = 4 .

the equation becomes ;

x² + 2*x*4 + 4² = 3 + 4²

(x + 4)² = 3 + 16

(x + 4)² = 19

taking root on both the sides ,

we get ,

x + 4 = ±√19

So , x = -4 + √19    or   x = -4 - √19

Simplifying further ,

we get ,

x = 0.35889   or x = -8.35889

rounding to the nearest tenth we get

x = 0.4   or x = -8.4  .

Therefore , the solution of the given equation is x = 0.4 , x = -8.4  .

Learn more about Completing the Square here

https://brainly.com/question/1372511

#SPJ4

The mortgage amount of a new home 170,000. The bank required a 8.6% down payment.

What was the original price before the down payment was made?

Answers

Answer:  15,507.44.

Step-by-step explanation: To find the original price of the home before the down payment was made, you need to use the following formula:

Original price = Mortgage amount / (1 - Down payment percentage)

Plugging in the values given in the question, you get:

Original price = $170,000 / (1 - 8.6%)

The original price of the home before the down payment was made is $<<170000/(1-.086)=185507.44>>185,507.44.

This means that the down payment amount was $185,507.44 - $170,000 = 185507.44-170000=15,507.44.

When a company sales x units of a product, its profits P can be determined
using the profit function
P(x) =
-2x² + 40x + 280
a) How many units must be sold to maximize profit?
b) What is the maximum profit?

Answers

(a) There are 10 units must be sold to maximize profit.

(b) The maximum profit is 480.

what is profit?

A profit is an income that is given to the owner in an efficient market production process in accounting. The owner's primary interest in the income-formation process of market production is profit, which is a measure of profitability. Different profit metrics are frequently employed.

Given:

A company sales x units of a product, its profits P can be determined

using the profit function P(x) = -2x² + 40x + 280       ..(1)

We have to find the maximum profit.

Differentiate equation (1) w.r.t x

     P'(x) = -4x +40            ..(2)

-4x + 40 = 0

        4x = 40

          x = 10

Again differentiate equation (2) w.r.t x

P''(x) = -4 which is negative.

Hence, there are 10 units must be sold to maximize profit.

Now, to find the maximum profit.

Plug x = 10 in equation (1)

P(10) = -2(10)^2 + 40(10) + 280

        = -200 + 400 + 280

P(10) = 480

Hence, the maximum profit is 480.

To know more about profit, click on the link

https://brainly.com/question/19104371

#SPJ1

6 3 points F L(2x-48) 32 W Using the diagram above find the value of x and use it to find the measures of all the missing angles. X = type your answer..... m/FOW= m/FLW= m/OLW= = type your answer... type your answer... type your answer.....

Answers

The value of x is 80

The measure of angle FOW, m ∠FOW, is 80°

The measure of angle FLW, m ∠FLW, is 112°

The measure of angle OLW, m ∠OLW, is = 68°

Calculating the measure of angles

From the question, we are to determine the measure of the missing angles

From the given diagram, we can write that

(2x - 48)° = x° + 32°

Solve for x

(2x - 48)° = x° + 32°

2x° - 48° = x° + 32°

Subtract x° from both sides of the equation

2x° - x° - 48° = x° - x° + 32°

x°  - 48° = 32°

Add 48° to both sides of the equation

x° - 48° + 48° = 32° + 48°

x° = 32° + 48°

x° = 80°

Therefore,

x = 80

Calculating the m ∠FOW

m ∠FOW = x°

m ∠FOW = 80°

Calculating the m ∠FLW

m ∠FLW = (2x - 48)°

m ∠FLW = (2(80) - 48)°

m ∠FLW = (160 - 48)°

m ∠FLW = 112°

Calculating the m ∠OLW

m ∠OLW + m ∠FOW + m ∠OWL = 180° (Sum of angles in a triangle)

m ∠OLW + 80° + 32° = 180°

m ∠OLW + 112° = 180°

m ∠OLW = 180° - 112°

m ∠OLW = 68°

Hence, the measure of m ∠OLW is 68°

Learn more on Calculating the measure of angles here: https://brainly.com/question/25215131

#SPJ1

Find the volume of the composite solid (STEP BY STEP PLEASE) 15 POINTS

Answers

Answer: 226.19

Step-by-step explanation:

Using the formula for a cyclinder{ V=πr2h} and inputing the values given in the diagram, (the radius and height given) you get 28.27.

Then adding the cone's volume (V=πr2h/3) and inputing that values in the diagram (the radius and height) you get 197.92.

Adding those values you got from the cyclinder and cone together, you get the volume of the composite solid.

Refer to the photo taken.

write and find the general solution of the differential equation that models the verbal statement. evaluate the solution at the specified value of the independent variable. the rate of change of p is proportional to p. when t

Answers

When t = 3869, the rate of change of p is proportional to p.

What is differential equation?

A differential equation in mathematics is an equation that connects the derivatives of one or more unknown functions.

Applications often involve functions that reflect physical quantities, derivatives that depict the rates at which those values change, and a differential equation that establishes a connection between the three. Due to the prevalence of these relationships, differential equations are widely used in many fields, including engineering, physics, economics, and biology.

The primary focus of studying differential equations is on the solutions—the collection of functions that satisfy each equation—as well as the characteristics of those solutions.

Only the most basic differential equations can be solved using explicit formulas, but many features of a particular differential equation's solutions can be deduced without doing an exact calculation.

When a closed-form expression is used, it is common for.

According to our question-

The revised equation is then

P=5000exp(-0.0513t)

Find P at t=5 now.

P=5000 exp(-0.0513×5)

P=5000 ×exp(-0.25647) (-0.25647)

P=5000 × 0.77378

P=3868.89.

P = 3869. Roughly

Learn more about differential equation click here:

https://brainly.com/question/1164377

#SPJ4

2. A gas in a flexible container with volume 8L has a temperature of 400 K. If the temperature of the gas changes to 200 K, what is the expected new volume of the gas?
A. 2 L
B. 4 L
C. 16 k
D. 32 L

Answers

The volume of the gas becomes 4L. Option B is correct.

What is proportionality?

proportionality is defined as between two or more sets of values, and how these values are related to each other in the sense are they directly proportional or inversely proportional to each other.

Here,

From the ideal gas equation, we know that,

Volume is directly proportional to the temperature,

So,

V₁ / V₂ = T₁ / T₂

Where,

V₁ and T₁ are the initial pressure and temperature,

V₂ and T₂ are the final pressure and temperature.

Substitute the values,

8 / V₂ = 400 / 200

V₂  = 4

Thus, the volume of the gas becomes 4L. Option B is correct.

Learn more about proportionality here:
brainly.com/question/22620356

#SPJ1

During surgery, patient's circulatory system requires at least 50 milligrams of an anesthetic: The amount of anesthetic present hours after 70 milligrams of anesthetic administered given by the following T(t) 70(0.727)t (a) How much, to the nearest milligram, of the anesthetic is present in the patient's circulatory system 30 minutes after the anesthetic is administered? (b) How long_ to the nearest minute can the operation last if the patient does not recelve additional anesthetic?

Answers

63 mg of anesthetic is present in the patient’s circulatory system.

The operation can last for 20 minutes.

During surgery, a patient’s circulatory system requires at least 50 mg of an anesthetic. The amount of anesthetic present t hours after 70 mg of anesthetic is administered is given by the following,

T(t) = 70 × (0.727) ^t

Time = 1/3 hours

T (1/3) = 70 × (0.727) ^1/3

            = 70 × ∛ 0.727

             = 70 × 0.899176

            = 62.94232

To the nearest mg: 63mg

Now,

63 = 70 × (0.727) ^t

63/70 = (0.727) ^t

0.9 = (0.727) ^t

Log0.9 = log (0.727) ^t

Log0.9 = t × log (0.727)  

T = log0.9 / log0.727

T = 0.330461

T = 0.330461 hours

  = 20 minutes

To know more about Logarithms, refer to:

http://brainly.com/question/25710806

#SPJ4

Determine which integer in the solution set will make the equation true

Answers

Answer:

2

Step-by-step explanation:

First, we must add 14 to both sides to put the variable and coefficient alone.

4s - 14 = -6

   +14     +14

4s = 8

s = 2

Other Questions
cost savings that come from learning by doing is referred to as: a. first-mover advantage. b. economies of scale. c. factor endowments. d. production possibilities frontier e. learning effects. Which of the following musical elements did the German Classical symphonists contribute to the symphony genre? hubo un buen programa en la televisn anoche? s. hubo tres programas interesantes. s. han tres programas interesantes. s. fueron tres programas interesantes. a method for political parties to select their candidates for office whereby party members convene at local meetings is referred to as Is Santa in Thailand? Find the measure of What form of government is run by a leader with absolute power? What is the difference between a direct democracy and a republic?What is ethnocentrism?What is culture shock? a tea shop sells 42 varieties of boxes of tea. iroh purchases 5 boxes of tea. how many ways are there for iroh to make a selection if there is no restriction on the varieties of tea purchased from the shop? under nasaa rules, which of the following records must be retained by investment advisers? i record of each written communication received by the investment adviser relating to any recommendation or advice ii record of each written communication sent by the investment adviser relating to any recommendation or advice iii copy of each notice, circular, advertisement, newspaper article, investment letter, bulletin or other communication (including electronic communications) sent to 2 or more persons. iv record of each securities transaction except for transaction not directed or controlled by the adviser and transactions in u.s. government securities. criminal codes: a. define the scope of offenses minors can commit. b. define the scope of offenses against defendants. c. define the scope of offenses against the public. d. define the scope of defenses against the public. 2a. . which country has a trade deficit and which has a trade surplus? explain how you got your answer and calculate the value of each ( /5) Transport layer protocols do not provide ___________ . A) logical communication between hosts B) delay guaranteesC) bandwidth guarantees D) All of the above responses are correct.D) All of the above responses are correct. in san diego, 150 people are willing to work an hour as secretaries if the wage is $20 per hour. for each additional $5 that the wage rises above $20, an additional 50 people are willing to work an hour as secretaries. for wages of $20, $25, $30, $35, and $40 per hour, use the orange points (square symbol) to plot the daily labor supply curve for secretaries on the following graph. supply 0 50 100 150 200 250 300 350 400 450 500 50 45 40 35 30 25 20 15 10 5 0 wage (dollars per hour) labor (number of workers) continue without saving an anthropologist finds a fossil in africa that appeared less than 100,000 years ago. to which species could the specimen belong?homo neanderthalensishomo habilishomo erectushomo sapiens There are several valid ways to represent an organic structure In a structural formula, all atoms and bonds are shown. For example, consider molecule A. Draw molecule A exactly as it appears in the image. Select Draw Rings More Erase Molecule in victorian times and in some ways even today, men who visited prostitutes or kept mistresses were accepted in polite society, but women who engaged in similar behavior were shunned. this is known as a when the base and leg side of right triangle are the same what is the angel measure fo the hypotenuse? which statement best describes the relationship between small independent music labels and the major music labels? in this water supply system, areas of need are removed from the water source, perhaps because of a higher elevation or remote location. the pumps in the system provide water directly to the system as well as pumping water into storage tanks. by filling the tanks at times of low use, the pumps do not have to provide the total supply to the system at times of high use I the ytem of equation conitent and independent conitent and dependent or inconitent Y=-x12y=-2x2